www.vorkurse.de
Ein Projekt von vorhilfe.de
Die Online-Kurse der Vorhilfe

E-Learning leicht gemacht.
Hallo Gast!einloggen | registrieren ]
Startseite · Mitglieder · Teams · Forum · Wissen · Kurse · Impressum
Forenbaum
^ Forenbaum
Status Mathe-Vorkurse
  Status Organisatorisches
  Status Schule
    Status Wiederholung Algebra
    Status Einführung Analysis
    Status Einführung Analytisc
    Status VK 21: Mathematik 6.
    Status VK 37: Kurvendiskussionen
    Status VK Abivorbereitungen
  Status Universität
    Status Lerngruppe LinAlg
    Status VK 13 Analysis I FH
    Status Algebra 2006
    Status VK 22: Algebra 2007
    Status GruMiHH 06
    Status VK 58: Algebra 1
    Status VK 59: Lineare Algebra
    Status VK 60: Analysis
    Status Wahrscheinlichkeitst

Gezeigt werden alle Foren bis zur Tiefe 2

Navigation
 Startseite...
 Neuerdings beta neu
 Forum...
 vorwissen...
 vorkurse...
 Werkzeuge...
 Nachhilfevermittlung beta...
 Online-Spiele beta
 Suchen
 Verein...
 Impressum
Das Projekt
Server und Internetanbindung werden durch Spenden finanziert.
Organisiert wird das Projekt von unserem Koordinatorenteam.
Hunderte Mitglieder helfen ehrenamtlich in unseren moderierten Foren.
Anbieter der Seite ist der gemeinnützige Verein "Vorhilfe.de e.V.".
Partnerseiten
Weitere Fächer:

Open Source FunktionenplotterFunkyPlot: Kostenloser und quelloffener Funktionenplotter für Linux und andere Betriebssysteme
Forum "Uni-Lineare Algebra" - Eigenvektoren Beweis
Eigenvektoren Beweis < Lineare Algebra < Hochschule < Mathe < Vorhilfe
Ansicht: [ geschachtelt ] | ^ Forum "Uni-Lineare Algebra"  | ^^ Alle Foren  | ^ Forenbaum  | Materialien

Eigenvektoren Beweis: Korrekturlesen
Status: (Frage) beantwortet Status 
Datum: 10:19 Mi 02.02.2005
Autor: muli

Hallo Ich habe hier einen beweis versucht und würde gern mal wissen ob man das so machen kann oder ob das alles schrott ist was ich hier gemacht habe:
Also die Aufgabe ist folgende:
Es sei V ein K-Vektorraum und f,g: V->V  Endomorphismen.Beweisen oder Wiederlegen sie folgende Aussage:
Ist v [mm] \in [/mm] V ein Eigenvektor von f und g  dann ist v auch ein Eigenvektor von f+g
Mein Beweis:
aus der Vorraussetzung folgt f(v) [mm] =\lambda [/mm] v=g(v)  mit v [mm] \in [/mm] V
Da V ein Vektorraum folgt daraus v+v [mm] \in [/mm] V
setze a,b [mm] \in [/mm] V mit a=b und a+b = V
ich weiss das f und g Homomorph daraus folgt:
[mm] \lambda [/mm] v =f(v)=g(v)= f(a+b)= f(a)+f(b)= f(a)+ g(b)= f(a)+ g(a)      q.e.d.
Das wars  ich hoffe  ihr könnt mir sagen ob das so geht oder auch nicht!!

muli


        
Bezug
Eigenvektoren Beweis: bitte KorrekturLesen
Status: (Mitteilung) Reaktion unnötig Status 
Datum: 11:09 Mi 02.02.2005
Autor: DaMenge

Hallo,

ich antworte auch mal...

Folgende Probleme sehe ich bei deinem Beweis:

>  Mein Beweis:
>  aus der Vorraussetzung folgt f(v) [mm]=\lambda[/mm] v=g(v)  mit v
> [mm]\in[/mm] V

Nein, es steht nicht da, dass es Eigenvektoren zum selben Eigenwert sein müssen. Damit ist der gesamte Beweis natürlich nicht richtig..

>  Da V ein Vektorraum folgt daraus v+v [mm]\in[/mm] V
>  setze a,b [mm]\in[/mm] V mit a=b und a+b = V

a+b=v ?!? das ist ein wenig problematisch - ich weiß gar nicht, ob das allgemein geht...

Es geht jedoch einfacher:
es gilt $ (f+g)(v)=f(v)+g(v) $ (das wird normaler Weise im Skript definiert o.ä.)
also : $ (f+g)(v)=f(v)+g(v) = [mm] \lambda_1 [/mm] *v [mm] +\lambda_2 [/mm] *v = [mm] (\lambda_1 +\lambda_2 [/mm] )*v $
und damit ist v wieder Eigenvektor...

viele Grüße
DaMenge


Bezug
        
Bezug
Eigenvektoren Beweis: Antwort
Status: (Antwort) fertig Status 
Datum: 11:19 Mi 02.02.2005
Autor: pjoas

Vorsicht, da sind einige Fallen dabei...

erstens, heisst es nur, dass v zufälligerweise Eigenvektor von f und g ist, das bedeutet noch lange nicht, dass die Eigenwerte gleich sind (was sie auch im Allgemeinen dann auch nicht sind!).
Also

$f(v) = [mm] \lambda{v}$ [/mm] und
$g(v) [mm] =\gamma{v}$ [/mm]

für die entsprechenden Matrizen (A für f und B für g) sähen die Gleichungen dann wie folgt aus:

[mm] $(A-\lambda{Id})v [/mm] = 0$
[mm] $(B-\gamma{Id})v [/mm] = 0$

addiert man diese erhält man:

[mm] $((A+B)-(\lambda+\gamma)Id)v=0$ [/mm]

also wird [mm] $\lambda+\gamma$ [/mm] Eigenwert zu $A+B$ mit Eigenvektor v.

Wie sieht es weiter mit f und g aus?

[mm] $f(v)+g(v)=\lambda{v}+\gamma{v} [/mm] = [mm] (\lambda [/mm] + [mm] \gamma)v [/mm] = (f+g)(v)$ ??? gilt dies und wenn: warum?

Fragen über Fragen, hoffe, es hilft dir weiter und hoffentlich hab ich nicht nur Unsinn erzählt - Fakt ist, dass du deinen Beweis ein wenig überarbeiten musst ;)

Gruß, Patrick


Bezug
Ansicht: [ geschachtelt ] | ^ Forum "Uni-Lineare Algebra"  | ^^ Alle Foren  | ^ Forenbaum  | Materialien


^ Seitenanfang ^
www.vorkurse.de
[ Startseite | Mitglieder | Teams | Forum | Wissen | Kurse | Impressum ]